Зображення сторінки
PDF
ePub
[blocks in formation]

du

[blocks in formation]

Now if the expression for be multiplied by (x+2)1

dx

and divided by (x+3), which are both essentially positive quantities, the result will be equal to

[merged small][merged small][merged small][merged small][merged small][merged small][merged small][ocr errors][merged small][merged small][merged small][merged small][merged small][merged small][merged small][merged small][merged small][merged small][merged small][merged small][merged small][merged small][merged small][ocr errors][merged small][merged small][merged small][merged small][merged small][merged small][merged small][merged small][merged small][merged small][merged small][merged small][merged small][merged small][ocr errors][merged small][merged small]

This is the solution of the problem, To find the height at which a light should be placed so that a small plane surface at a given horizontal distance shall receive the greatest illumination from it.

[ocr errors][merged small][merged small][merged small][merged small][merged small][merged small][merged small][merged small][merged small]

This is a solution of the problem, To find the magnitude of the body which must be interposed between two others so that the velocity communicated from the one to the other shall be a maximum.

[merged small][merged small][merged small][merged small][merged small][merged small][merged small][merged small][ocr errors][ocr errors][ocr errors][merged small][merged small][merged small][merged small][merged small][merged small][merged small][merged small][merged small]

This is a solution of the problem, To find in what direction a body must be projected with a given velocity that its range on a given plane may be the greatest possible.

J (13)

u =

(sin mx)9

(sin a)› m being an integer.

The values of a derived from m tan≈ = tan mæ, make u a maximum.

The values of a derived from sin mx = 0, make u a minimum.

The values of a derived from sin a = 0, make u a maximum and equal to m2.

[merged small][merged small][merged small][merged small][merged small][merged small][merged small][ocr errors][merged small][merged small][merged small][merged small][merged small][merged small]
[merged small][merged small][merged small][merged small][merged small][merged small][merged small][merged small][merged small][merged small][merged small][merged small][merged small][merged small][merged small][merged small][merged small][ocr errors][merged small][merged small][merged small][merged small][merged small][merged small][merged small]

hence, multiplying by § (x − a)1 which is essentially positive,

[merged small][merged small][merged small][merged small][merged small][merged small][ocr errors]

Therefore if c be positive xa makes u a minimum ; and if c be negative, a maximum.

(19)

du

dx

[blocks in formation]

Since = 1⁄2 c (x − a)3, a quantity insusceptible of a

change of sign, it appears that a which makes

gives neither a maximum nor a minimum.

[ocr errors]

(20)

u = (1 + x3) (7 − x)2.
x=7 gives u a minimum;

x = 1 gives u a maximum;

x = 0 gives u a minimum.

[blocks in formation]

(21) To divide a number a into a number of equal parts

such that their continued product shall be a maximum.

α

Let a be the number of parts, then is one of the parts,

20

a

a

[ocr errors]

and is the continued product, which is to be a maximum.

[merged small][merged small][merged small][merged small][merged small][merged small][merged small][merged small][merged small][merged small][ocr errors][ocr errors][ocr errors][merged small][merged small][merged small][merged small]

(22) From two points A, B (fig. 1), to draw two straight lines to a point P in a given line ON such that AP + BP shall be a minimum.

Take the given line as the axis of x, O the origin.

Let OP= x, and let the co-ordinates of A and B be a, b, a1, b1. Then

u = AP + BP = {b2 + (x − a)3} } + {b‚2 + (a ̧ − x)2 } 3 = minimum.

Whence

[ocr errors]

=

2

a1 <-00

{b2 + (x − a)2} } = {b‚2 + (a ̧ − x)2} } '

or the angles APM, BPN are equal.

[ocr errors]

(23) To find the point in the straight line AD (fig. 2), at which BC subtends the greatest angle; ABC being perpendicular to AD.

If the angle be a maximum its tangent is also a maximum.

Let P be the point, AP = x, AC = a,

[blocks in formation]

AB = b;

(a - b) x

ab + x2

is to be a maximum.

therefore

[blocks in formation]

;

(24) Through a point M (fig. 3) within the angle BAC draw the line PQ so that the triangle PAQ shall be a mini

mum.

Draw MN parallel to AQ, and let AN= a, MN = b, AP. Then 2a makes PAQ a minimum, and PQ is

bisected in M.

(25) Given the length of the arc of a circle, find the angle which it must subtend at the centre in order that the corresponding segment may be a maximum.

Let a be the half-length of the arc, w the radius of the

a

circle; then is the half-angle of the segment.

[merged small][merged small][merged small][merged small][merged small][merged small][merged small][merged small][merged small][merged small][ocr errors][merged small][merged small][ocr errors][merged small][merged small][merged small][merged small][merged small][merged small][merged small][ocr errors][merged small][merged small][merged small][ocr errors][merged small][merged small][merged small][ocr errors][merged small][merged small][merged small][merged small][merged small][ocr errors][merged small][merged small][merged small][merged small][merged small][merged small][merged small][ocr errors][merged small][merged small]

excluded by the geometry of the problem.

A geometrical solution of this problem is given in the Mathematical Collections of Pappus, Book V. Theor. 16.

(26) AC (fig. 4) and BD being parallel, it is required to draw from C a line CXY, such that the sum of the triangles ACX and BXY shall be a minimum.

If AC = a, AB = b, AX = ∞, it is easily seen that the area of the triangle ACX is proportional to ax, and that of a (b − x)2

BXY to

[ocr errors][ocr errors]

so that we have

[blocks in formation]

Vincent Viviani, Geometrica Divinatio, p. 152.

« НазадПродовжити »